You are on page 1of 38

Downloaded From : www.EasyEngineering.

net

GATE SOLVED PAPER - EE


2013

Q.No.1-25 Carry One Mark Each

Q. 1 Given a vector field Fv = y2 xavx - yzavy - x2 avz , the line integral # Fv : dlv evaluated
(A) - 2.33
et
along a segment on the x -axis from x = 1 to x = 2 is
(B) 0
(C) 2.33
.n (D) 7
Sol. 1

ng
Hence (B) is correct option

ri
Given, the field vector
Fv = y2 xavx - yzavy - x2 avz

ee
For the line segment along x -axis, we have
dlv = dxavx
So, v
in
F : dlv = ^y2 x h^dx h

ng
Since, on x -axis y = 0 so,
Fv : dlv = 0
or, #
yeFv : dlv = 0
2 - 2 x1 0
Q. 2

as
The equation >
1 - 1H>x2H >0H
= has
x1 0
(B) only one solution > H = > H
.E
(A) no solution
x2 0

w
(C) non-zero unique solution (D) multiple solutions
Sol. 2

w
Hence (D) is correct option
Given the equations in matrix form as
w >
2 - 2 x1 0
H> H = > H
1 - 1 x2 0
So, it is a homogenous set of linear equation. It has either a trivial solution
^x1 = x2 = 0h or an infinite no. of solution. Since, for the matrix
2 -2
A => H
1 -1
we have the determinant
A =0
Hence, it will have multiple solutions
Q. 3 Square roots of - i , where i = - 1 , are
(A) i , - i
(B) cos d- p n + i sin d- p n, cos b 3p l + i sin b 3p l
4 4 4 4

(C) cos d p n + i sin b 3p l, cos b 3p l + i sin d p n


4 4 4 4
(D) cos b 3p l + i sin b- 3p l, cos b- 3p l + i sin b 3p l
4 4 4 4

Downloaded From : www.EasyEngineering.net


Downloaded From : www.EasyEngineering.net

GATE SOLVED PAPER - EE 2013

Sol. 3 Hence (B) is correct option


We know that
i = eip/2 (In phasor form)
or, - i = e-ip/2
- i = !^e-ip/2h = ! e-ip/4
1/2
So,
= !=cos d p n - i sin d p nG
4 4
= cos d p n - i sin d p n ; - cos d p n + i sin d p n

= 1
et 4
- i ; -1 +
4
i
4 4

.n2 2 2

g
This is equivalent to the given option (B) only.

n
i
Q. 4 Three moving iron type voltmeters are connected as shown below. Voltmeter

readings is
er
readings are V , V1 and V2 as indicated. The correct relation among the voltmeter

ne
gi
en
sy
(A) V = V1 + V2 (B) V = V1 + V2
2
a 2

.E
(C) V = V1 V2 (D) V = V2 - V1
Sol. 4
w
Hence (B) is correct option

w
For an ideal voltmeter interval resistance is always zero. So we can apply the KVL
along the two voltmeters as

or
w V - V1 - V2 = 0
V = V1 + V2

Q. 5 Leakage flux in an induction motor is


(A) flux that leaks through the machine
(B) flux that links both stator and rotor windings
(C) flux that links none of the windings
(D) flux that links the stator winding or the rotor winding but not both
Sol. 5 Hence (D) is correct option
Leakage flux in an induction motor is flux that linked the stator winding or the
rotor winding but not both.

Downloaded From : www.EasyEngineering.net


Downloaded From : www.EasyEngineering.net

GATE SOLVED PAPER - EE 2013

Q. 6 The angle d in the swing equation of a synchronous generator is the


(A) angle between stator voltage and current
(B) angular displacement of the rotor with respect to the stator
(C) angular displacement of the stator mmf with respect to a synchronously
rotating axis.
(D) angular displacement of an axis fixed to the rotor with respect to a
synchronously rotating axis
Sol. 6 Hence (D) is correct option
The angle d in the swing equation of a synchronous generator is the angular

axis.
et
displacement of an axis fixed to the rotor with respect to a synchronously rotating

Q. 7

.n
Consider a delta connection of resistors and its equivalent star connection as

g
shown below. If all elements of the delta connection are scaled by a factor k , k > 0

n
, the elements of the corresponding star equivalent will be scaled by a factor of

ri
ee
(A) k2 in
g
(B) k

Sol. 7
(C) 1/k

en
Hence (B) is correct option
(D) k

y
In the equivalent star connection, the resistance can be given as
s
a RC = Rb Ra
Ra + Rb + Rc

.E RB = Ra Rc
Ra + Rb + Rc

w RA = Rb Rc

w Ra + Rb + Rc
So, if the delta connection components Ra , Rb and Rc are scaled by a factor k
w
then
^k Rb h^k Rc h
RAl =
kRa + kRb + kRc
2
=k Rb Rc
k Ra + Rb + Rc
= k RA
Hence, it is also scaled by a factor k
Q. 8 A band-limited signal with a maximum frequency of 5 kHz is to be sampled.
According to the sampling theorem, the sampling frequency which is not valid is
(A) 5 kHz (B) 12 kHz
(C) 15 kHz (D) 20 kHz
Sol. 8 Hence (A) is correct option
Given, the maximum frequency of the band-limited signal
fm = 5 kHz
According to the Nyquist sampling theorem, the sampling frequency must be

Downloaded From : www.EasyEngineering.net


Downloaded From : www.EasyEngineering.net

GATE SOLVED PAPER - EE 2013

greater than the Nyquist frequency which is given as


fN = 2fm = 2 # 5 = 10 kHz
So, the sampling frequency fs must satisfy
fs $ fN
fs $ 10 kHz
only the option (A) does not satisfy the condition therefore, 5 kHz is not a
valid sampling frequency.
Q. 9
et
For a periodic signal v ^ t h = 30 sin 100t + 10 cos 300t + 6 sin ^500t + p/4h, the

(A) 100
.n
fundamental frequency in rad/s

(B) 300
ng
(C) 500
ri
e
(D) 1500
Sol. 9

ne
Hence (A) is correct option

i
Given, the signal
v ^ t h = 30 sin 100t + 10 cos 300t + 6 sin ^500t + p4 h
So we have
ng
ye w1 = 100 rad/s
w2 = 300 rad/s

as w3 = 500 rad/s

.E
Therefore, the respective time periods are
T1 = 2p = 2p sec

w w1 100
T2 = 2p = 2p sec
w w2 300

w T3 = 2p sec
500
So, the fundamental time period of the signal is
LCM ^2p, 2p, 2ph
L.C.M. ^T1, T2 T3h =
HCF ^100, 300, 500h
or, T0 = 2p
100
Thus, the fundamental frequency in rad/sec is
w0 = 2p = 100 rad/s
10

Q. 10 A bulb in a staircase has two switches, one switch being at the ground floor
and the other one at the first floor. The bulb can be turned ON and also can
be turned OFF by any one of the switches irrespective of the state of the other
switch. The logic of switching of the bulb resembles
(A) and AND gate
(B) an OR gate
(C) an XOR gate
(D) a NAND gate

Downloaded From : www.EasyEngineering.net


Downloaded From : www.EasyEngineering.net

GATE SOLVED PAPER - EE 2013

Sol. 10 Let A denotes the position of switch at ground floor and B denotes the position
of switch at upper floor. The switch can be either in up position or down position.
Following are the truth table given for different combinations of A and B
A B Y(Bulb)
up(1) up(1) OFF(0)
Down(0) Down(0) OFF(0)
up(1) Down(0) ON(1)
Down(0) up(1) ON(1)
When the switches A and B are both up or both down, output will be zero (i.e.

et
Bulb will be OFF). Any of the switch changes its position leads to the ON state
of bulb. Hence, from the truth table, we get

.n
Y = A5B
i.e., the XOR gate

ng
Hence (C) is correct option
Q. 11
ri
The Bode plot of a transfer function G ^s h is shown in the figure below.

ee
in
ng
ye
as
.E
w
w
The gain _20 log G ^s h i is 32 dB and - 8 dB at 1 rad/s and 10 rad/s respectively.
w
The phase is negative for all w. Then G ^s h is
(A) 39.8 (B) 392.8
s s

(C) 32 (D) 322


s s
Sol. 11 Hence (B) is correct option
From the given plot, we obtain the slope as
20 log G2 - 20 log G1
Slope =
log w2 - log w1
From the figure
20 log G2 =- 8 dB
20 log G1 = 32 dB
and w1 = 1 rad/s
w2 = 10 rad/s
So, the slope is
Slope = - 8 - 32
log 10 - log 1

Downloaded From : www.EasyEngineering.net


Downloaded From : www.EasyEngineering.net

GATE SOLVED PAPER - EE 2013

=- 40 dB/decade
Therefore, the transfer function can be given as
G ^s h = k2
S
at w = 1
G ^ jwh = k 2 = k
w
In decibel,
20 log G ^ jwh = 20 log k = 32
or,
et 32

k = 10 = 39.8
Hence, the Transfer function is
20

.n
G ^s h = k2 = 392.8
s s

ng
Q. 12

ri
In a voltage-voltage feedback as shown below, which one of the following statements
is TRUE if the gain k is increased?

ee
in
ng
ye
as
.E
(A) The input impedance increases and output impedance decreases

w
(B) The input impedance increases and output impedance also increases

w
(C) The input impedance decreases and output impedance also decreases
(D) The input impedance decreases and output impedance increases

Sol. 12
w
Hence (A) is correct option
The i/p voltage of the system is given as
Vin = V1 + Vf
= V1 + k Vout
= V1 + k A 0 V1 ^Vout = A 0 V1h
= V1 ^1 + k A 0h
Therefore, if k is increased then input voltage is also increased so, the input
impedance increases. Now, we have
Vout = A 0 V1
= A0 Vin
^1 + k A 0h
= A 0 Vin
^1 + k A 0h
Since, Vin is independent of k when seen from output mode, the output voltage
decreases with increase in k that leads to the decrease of output impedance.
Thus, input impedance increases and output impedance decreases.

Downloaded From : www.EasyEngineering.net


Downloaded From : www.EasyEngineering.net

GATE SOLVED PAPER - EE 2013

Q. 13 The input impedance of the permanent magnet moving coil (PMMC) voltmeter is
infinite. Assuming that the diode shown in the figure below is ideal, the reading
of the voltmeter in Volts is

et
(A) 4.46
.n (B) 3.15
(C) 2.23

ng (D) 0
Sol. 13

ri
Hence (A) is correct option
For the + ve half cycle of I/p voltage, diode will be forward biased (Vg = 0 , ideal

ee
diode) Therefore, the voltmeter will be short circuited and reads
V1 = 0 volt (for + ve half cycle)

in
Now, for - ve half cycle, diode will be reverse biased and treated as open

ng
circuit. So, the voltmeter reads the voltage across 100 kW. Which is given by
14.14 0c

So, ye V2 = 100 #

V2,rms = 14 volt
100 + 1

as 2
Therefore, the average voltage for the whole time period is obtained as
V + V2, rms 0 + ^14/ 2 h
.E Vave = 1
2
=
2
= 14
2 2

w = 4.94 . 4.46 volt


Q. 14
w
The curl of the gradient of the scalar field defined by V = 2x2 y + 3y2 z + 4z2 x is
(A) 4xyavx + 6yzavy + 8zxavz
w
(B) 4avx + 6avy + 8avz
(C) ^4xy + 4z2h avx + ^2x2 + 6yz h avy + ^3y2 + 8zx h avz
(D) 0
Sol. 14 Hence (D) is correct option
Given the scalar field
V = 2x2 y + 3y2 z + 4z2 x
Its gradient is given by
dV = ^4xy + 4z2h avx + _2x2 + 6yz i avy + ^3y2 + 8zx h avz
So, the curl of the gradient is obtained as
avx avy avz
d # ^dvh = 2
rx
2
ry
2
rz
2 2 2
4xy + 4z 2z + 6yz 3y + 8zx
= avx ^6y - 6y h - avy ^8z - 8z h + avz ^4x - 4x h
=0
Note : From the properties of curl, we know that curl of gradient of any scalar
field is always zero. So, there is no need to solve the curl and gradient.

Downloaded From : www.EasyEngineering.net


Downloaded From : www.EasyEngineering.net

GATE SOLVED PAPER - EE 2013

Q. 15 A continuous random variable X has a probability density function f ^x h = e-x ,


0 < x < 3. Then P "X > 1, is
(A) 0.368 (B) 0.5
(C) 0.632 (D) 1.0
Sol. 15 Hence (A) is correct option
Given, the PdF of random variable x as
f ^x h = e-x 0<x<3
So,

e
P ^x > 1h =
t #e
3-x
dx

.n 1
-x 3
= :e D

ng -1 1
= e-1

ri = 0.368
Q. 16

ee
The flux density at a point in space is given by Bv = 4xavx + 2kyavy + 8avz Wb/m2 .
The value of constant k must be equal to
(A) - 2
in (B) - 0.5

Sol. 16
(C) + 0.5

ng
Hence (A) is correct option
(D) + 2

ye
Given, flux density
Bv = 4x avx + 2ky avy + 8 avz

as
Since, magnetic flux density is always divergence less.
d$Bv = 0

.E
i.e.,
So, for given vector flux density, we have
d$Bv = 4 + 2k + 0 = 0
or, w k =- 2
Q. 17
w
A single-phase transformer has no-load loss of 64 W, as obtained from an open

w
circuit test. When a short-circuit test is performed on it with 90% of the rated
currents flowing in its both LV and HV windings, he measured loss is 81 W. The
transformer has maximum efficiency when operated at
(A) 50.0% of the rated current (B) 64.0% of the rated current
(C) 80.0% of the rated current (D) 88.8% of the rated current
Sol. 17 Hence (C) is correct option
Since, the no-load loss (when load is not connected) is 64 W. So, the open
circuit loss is
W0 = 6 watt (no load)
As the 90% of rated current flows in its both LV and HV windings, the lost is
measured as
copper loss at 90% load = 81 watt
or, Wcu ^at 90%h = 81 watt
Now, we have the copper loss for x load as
2
Wcu ^x h = Wcu ^at full loadh # c Ix m
IFL
2
or, Wcu ^at full loadh = Wcu ^at x% loadh # c IFL m
Ix
= ^81h # 1
^0.9h2

Downloaded From : www.EasyEngineering.net


Downloaded From : www.EasyEngineering.net

GATE SOLVED PAPER - EE 2013

= 100 watt
For maximum efficiency we must have
copper lossed = no-load loss
= 64 watt
Consider at x% load copper loss is 64 watt. So,
^x2h Wcu ^at full loadh = 64
or, x = 64 = 0.8
100
or x = 80% load
Q. 18

et
A single-phase load is supplied by a single-phase voltage source. If the current
flowing from the load to the source is 10+ - 150cA and if the voltage at the load

.n
terminal is 100+60cV , then the
(A) load absorbs real power and delivers reactive power
g
(B) load absorbs real power and absorbs reactive power
n
i
(C) load delivers real power and delivers reactive power
r
Sol. 18
ee
(D) load delivers real power and absorbs reactive power
Hence (B) is correct option

in
Consider the voltage source and load shown in figure

ng
ye
as
.E
w
We obtain the power delivered by load as

w Pdelivered = I L* VL
= ^10 + 150ch^10 60ch
w = 100 210c
= 1000 cos 210c + j1000 sin 210c
=- 866.025 - j500
As both the reactive and average power (real power) are negative so, power is
absorbed by load. i.e., load absorbs real as well as reactive power.
Q. 19 A source vs ^ t h = V cos 100pt has an internal impedance of ^4 + j3h W . If a purely
resistive load connected to this source has to extract the maximum power out of
the source, its value in W should be
(A) 3 (B) 4
(C) 5 (D) 7
Sol. 19 Hence (C) is correct option
For the purely resistive load, maximum average power is transferred when
2 2
RL = RTh + XTh
where RTh + jXTh is the equivalent thevinin (input) impedance of the circuit.
i.e., RL = 42 + 32
= 5W

Downloaded From : www.EasyEngineering.net


Downloaded From : www.EasyEngineering.net

GATE SOLVED PAPER - EE 2013

Q. 20 Two systems with impulse responses h1 ^ t h and h2 ^ t h are connected in cascade.


Then the overall impulse response of the cascaded system is given by
(A) product of h1 ^ t h and h2 ^ t h
(B) sum of h1 ^ t h and h2 ^ t h
(C) convolution of h1 ^ t h and h2 ^ t h
(D) subtraction of h2 ^ t h from h1 ^ t h
Sol. 20 Hence (C) is correct option

et
If the two systems with impulse response h1 ^ t h and h2 ^ t h are connected in
cascaded configuration as shown in figure, then the overall response of the system

.n
is the convolution of the individual impulse responses.

ng
ri
ee
in
g
Q. 21 Which one of the following statements is NOT TRUE for a continuous time
causal and stable LTI system?
n
(A) All the poles of the system must lie on the left side of the jw axis
e
sy
(B) Zeros of the system can lie anywhere in the s-plane
(C) All the poles must lie within s = 1

a
(D) All the roots of the characteristic equation must be located on the left side

.E
of the jw axis.

w
Sol. 21 Hence (C) is correct option
For a system to be casual, the R.O.C of system transfer function H ^s h which is

w
rational should be in the right half plane and to the right of the right most pole.

w For the stability of LTI system. All poles of the system should lie in the
left half of S -plane and no repeated pole should be on imaginary axis. Hence,
options (A), (B), (D) satisfies both stability and causality an LTI system.
But, Option (C) is not true for the stable system as, S = 1 have one pole
in right hand plane also.
Q. 22 The impulse response of a system is h ^ t h = tu ^ t h. For an input u ^t - 1h, the
output is
2 t ^t - 1h
(A) t u ^ t h (B) u ^t - 1h
2 2

^t - 1h2 2
(C) u ^t - 1h (D) t - 1 u ^t - 1h
2 2
Sol. 22 Hence (C) is correct option
Given, the input
x ^ t h = u ^t - 1h
It’s laplace transform is
-s
X ^s h = e
s
The impulse response of system is given

Downloaded From : www.EasyEngineering.net


Downloaded From : www.EasyEngineering.net

GATE SOLVED PAPER - EE 2013

h^t h = t u^t h
Its Laplace transform is
H ^s h = 12
s
Hence, the overall response at the output is
Y ^s h = X ^s h H ^s h
-s
=e3
s
its inverse laplace transform is
^t - 1h2
y^t h =

et 2
u ^t - 1h

.n
Q. 23

ng
Assuming zero initial condition, the response y ^ t h of the system given below to a
unit step input u ^ t h is

ri
ee
in
(A) u ^ t h
2
n
(C) t u ^ t h
g (B) tu ^ t h
(D) e-t u ^ t h
2

ye
Sol. 23

as
Hence (B) is correct option
The Laplace transform of unit step fun n is

.E U ^s h = 1
s

w
So, the O/P of the system is given as

w Y ^s h = b 1 lb 1 l
s s

w = 12
s
For zero initial condition, we check
dy ^ t h
u^t h =
dt
& U ^s h = SY ^s h - y ^0 h
& U ^s h = s c 12 m - y ^0 h
s
or, U ^s h = 1
^y ^0 h = 0h
s
Hence, the O/P is correct which is
Y ^s h = 12
s
its inverse Laplace transform is given by
y ^ t h = tu ^ t h

Downloaded From : www.EasyEngineering.net


Downloaded From : www.EasyEngineering.net

GATE SOLVED PAPER - EE 2013

V2 ^s h
Q. 24 The transfer function of the circuit shown below is
V1 ^s h

(A) 0.5s + 1
et (B) 3s + 6

n
s+1 s+2
(C) s + 2
s+1
g. (D) s + 1
s+2
Sol. 24
in
Hence (D) is correct option

er
For the given capacitance, C = 100mF in the circuit, we have the reactance.
XC = 1

ne sc

i = 1
s # 100 # 10-6

ng = 10
s
4

So,

ye V2 ^s h
10 4 + 10 4

as V1 ^s h
= 4 s
10 + 10 4 + 10 4
s s

.E = s +
s+2
1

w
Q. 25
w
In the circuit shown below what is the output voltage ^Vouth if a silicon transistor

w
Q and an ideal op-amp are used?

(A) - 15 V (B) - 0.7 V


(C) + 0.7 V (D) + 15 V
Sol. 25 Hence (B) is correct option
For the given ideal op-amp, negative terminal will be also ground (at zero voltage)
and so, the collector terminal of the BJT will be at zero voltage.
i.e., VC = 0 volt
The current in 1 kW resistor is given by
I = 5 - 0 = 5 mA
1 kW

Downloaded From : www.EasyEngineering.net


Downloaded From : www.EasyEngineering.net

GATE SOLVED PAPER - EE 2013

This current will flow completely through the BJT since, no current will flow
into the ideal op-amp ( I/P resistance of ideal op-amp is infinity). So, for BJT
we have
VC = 0
VB = 0
IC = 5 mA
i.e.,the base collector junction is reverse biased (zero voltage) therefore, the
collector current (IC ) can have a value only if base-emitter is forward biased.
Hence,

&
et
VBE = 0.7 volts
VB - VE = 0.7
&
or, n
0 - Vout = 0.7
.
Vout =- 0.7 volt
Q. 26
ng
When the Newton-Raphson method is applied to solve the equation
i
f ^x h = x3 + 2x - 1 = 0 , the solution at the end of the first iteration with the initial
r
e
value as x 0 = 1.2 is

e
(A) - 0.82 (B) 0.49
(C) 0.705
in (D) 1.69
Sol. 26

ng
Hence (C) is correct option
Given, the equation

ye f ^x h = x3 + 2x - 1 = 0
as initial condition is x 0 = 1.2

as
so, from N - R method we obtain
f ^xn h

.E
xn + 1 = xn -
f l^xn h
here,
w x0 = 1.2
w f ^x 0h = ^1.23h + 2 ^1.2h - 1 = 3.128
w
Also,
So,
f l^x h
f l^x 0h
= 3x2 + 2
= 3 ^1.2h2 + 2 = 6.32
Hence, 1 st iterative value is
f ^x 0h
x1 = x 0 -
f l^x 0h
= 1.2 - 3.128
6.32
= 0.705
Q. 27 A function y = 5x2 + 10x is defined over an open interval x = ^1, 2h. Atleast at one
point in this interval, dy/dx is exactly
(A) 20 (B) 25
(C) 30 (D) 35
Sol. 27 Hence (B) is correct option
Given the function
y = f ^x h = 5x2 + 10x in the internal x = ^1, 2h

Downloaded From : www.EasyEngineering.net


Downloaded From : www.EasyEngineering.net

GATE SOLVED PAPER - EE 2013

Since, function y is continuous in the interval ^1, 2h as well as its is


differentiable at each point so, from Lagranges mean value theorem there exist
at least a point where
f ^b h - f ^a h
f l^c h =
b-a
Here, we have
a=1,b=2

t
So, for x = a = 1, we obtain
y = f ^a h = f ^1 h = 5 ^1 h2 + 10 ^1 h = 15
and for x = b = 2
ne
Therefore,
g.
y = f ^b h = f ^2 h = 5 ^2 h2 + 10 ^2 h = 40

in
f l^c h = 40 - 15 = 25
2-1
r
This is the exact value of dy/dx at point x = c in the interval x = ^1, 2h.
e
Q. 28

ne
A 4-pole induction motor, supplied by a slightly unbalanced three-phase 50 Hz
source, is rotating at 1440 rpm. The electrical frequency in Hz of the induced

(A) 100
gi
negative sequence current in the rotor is
(B) 98
(C) 52
en (D) 48
Sol. 28
Given,
sy
Hence (B) is correct option

a
frequency of source, f = 50 Hz

.E
no. of poles P =4

w
rotating speed N = 1440 rpm
Now, the synchronous speed is determined as
w NS =
120f

w =
P

4 ^ h
120 50 = 1500 rpm
So, the slip in the motor is
S = NS - N
NS
= 1500 - 1440 = 0.04
1500
Now, the electrical frequency of the induced negative sequence current in rotor
is obtained as
f0 = ^2 - 5h f
where f is stator frequency given as f = 50 Hz .
Therefore, f0 = ^2 - 0.04h 50 = 98 Hz
Q. 29 Thyristor T in the figure below is initially off and is triggered with a single
pulse of width 10 ms . It is given that L = b 100 l mH and C = b 100 l mF . Assuming
p p
latching and holding currents of the thyristor are both zero and the initial charge
on C is zero, T conducts for

Downloaded From : www.EasyEngineering.net


Downloaded From : www.EasyEngineering.net

GATE SOLVED PAPER - EE 2013

(A) 10 ms (B) 50 ms
(C) 100 ms (D) 200 ms
Sol. 29 Hence (C) is correct option
Given,

et
L = 100 mH
p
C =
.n100
p
mF

g
When the circuit is triggered by 10 ms pulse, the thyristor is short circuited and
n
so, we consider

ri
IC = Im sin wt

ee
Therefore, voltage stored across capacitor is
VC = 1 # IC dt

in C
= Vm ^1 - cos wt h

g
where w is angular frequency obtained as

n w = 1 = 1 = p # 10 4

ye LC b p l # 10
100 -6

So,

as T = 1 = 2p = 200 ms
f w

.E
As IC = Im sin wt oscillates between - ve and - ve half cycle so,
circuit is conducting for only half of cycle and thyristor is open after half cycle.
i.e.,w the conduction period = T = 100 ms
2
Q. 30
w
The following arrangement consists of an ideal transformer and an attenuator

w
which attenuates by a factor of 0.8. An ac voltage VWX1 = 100 V is applied
across WX to get an open circuit voltage VYZ1 across YZ. Next, an ac voltage
VYZ2 = 100 V is applied across YZ to get an open circuit voltage VWX2 across WX.
Then, VYZ1 /VWX1 , VWX2 /VYZ2 are respectively,

(A) 125/100 and 80/100 (B) 100/100 and 80/100


(C) 100/100 and 100/100 (D) 80/100 and 80/100
Sol. 30 Hence (C) is correct option
For the given transformer, we have
V = 1.25
VWX 1

Downloaded From : www.EasyEngineering.net


Downloaded From : www.EasyEngineering.net

GATE SOLVED PAPER - EE 2013

Since, VYZ = 0.8 (attenuation factor)


V
So,
t
^ h^ h
VYZ = 0.8 1.25 = 1
VWX
e
or,
.n
VYZ = VWX
V
VWX = 100 V ; YZ = 100
g
at 1
1
VWX 100

n
1

at
ri
VWZ = 100 V ;
2
VWX
VYZ
= 100
2

100

e
2

Q. 31

ne
Two magnetically uncoupled inductive coils have Q factors q1 and q2 at the chosen
operating frequency. Their respective resistances are R1 and R2 . When connected

(A) q1 + q2 gi
in series, their effective Q factor at the same operating frequency is
(B) ^1/q1h + ^1/q2h

en
(C) ^q1 R1 + q2 R2h / ^R1 + R2h (D) ^q1 R2 + q2 R1h / ^R1 + R2h
Sol. 31

sy
Hence (C) is correct option
The quality factor of the inductances are given by
a q 1 = wL 1

.E
R1
q 2 = wL 2
and
w R2

w
So, in series circuit, the effective quality factor is given by
XLeq
w Q =
Req
= wL 1 + wL 2
R1 + R 2
wL 1 + wL 2
= 1 R 2 R1 R 2
R
1 + 1
R 2 R1
q1 q
+ 2
R
= 2 R 2
1 + 1
R 2 R1
q1 R1 + q 2 R 2
=
R1 + R 2

Q. 32 The impulse response of a continuous time system is given by h ^ t h = d ^t - 1h + d ^t - 3h


. The value of the step response at t = 2 is
(A) 0 (B) 1
(C) 2 (D) 3
Sol. 32 Hence (B) is correct option
Given, the impulse response of continuous time system
h ^ t h = d ^t - 1h + d ^t - 3h

Downloaded From : www.EasyEngineering.net


Downloaded From : www.EasyEngineering.net

GATE SOLVED PAPER - EE 2013

From the convolution property, we know


x ^ t h * d ^t - t 0h = x ^t - t 0h
So, for the input
x ^ t h = u ^ t h (Unit step fun n )
The output of the system is obtained as
y^t h = u^t h * h^t h

= u ^ t h * 6d ^t - 1h + d ^t - 3h@

= u ^t - 1h + u ^t - 3h
at t = 2
t
y ^2 h = u ^2 - 1h + u ^2 - 3h
e
n
=1

Q. 33
g.
The signal flow graph for a system is given below. The transfer function
Y ^s h
U ^s h
for
this system is
in
er
ne
gi
(A)
en
s+1 (B) s+1

sy
5s2 + 6s + 2

s+1
s 2 + 6s + 2

(C)
a
s 2 + 4s + 2
(D) 2 1
5s + 6s + 2
Sol. 33

.E
Hence (A) is correct option
For the given SFG, we have two forward paths
w Pk1 = ^1 h^s-1h^s-1h^1 h = s-2
w Pk2 = ^1 h^s-1h^1 h^1 h = s-1

w
since, all the loops are touching to both the paths Pk1 and Pk2 so,
Dk 1 = Dk 2 = 1
Now, we have
D = 1 - (sum of individual loops)
+ (sum of product of nontouching loops)
Here, the loops are
L1 = ^- 4h^1 h =- 4
L2 = ^- 4h^s-1h = 4s-1
L 3 = ^- 2h^s-1h^s-1h =- 2s-2
L 4 = ^- 2h^s-1h^1 h =- 2s-1
As all the loop L1, L2, L 3 and L 4 are touching to each other so,
D = 1 - ^L1 + L2 + L 3 + L 4h
= 1 - ^- 4 - 4s-1 - 2s-2 - 2s-1h
= 5 + 6s-1 + 2s-2
From Mason’s gain formulae

Downloaded From : www.EasyEngineering.net


Downloaded From : www.EasyEngineering.net

GATE SOLVED PAPER - EE 2013

Y ^s h
= SPk Dk
U ^s h D
= s-2 + s-1
5 + 6s-1 + 2s-2
= 2s+1
5s + 6s + 2

Q. 34 In the circuit shown below the op-amps are ideal. Then, Vout in Volts is

et
.n
ng
ri
ee
in
ng
(A) 4
ye (B) 6
(C) 8
as (D) 10

.E
Sol. 34 Hence (C) is correct option

w
w
w

For the given ideal op-Amps we can assume


V 2- = V 2+ = V2 (ideal)
V 1+ = V 1- = V1 (ideal)
So, by voltage division
V1 = Vout # 1
2
Vout = 2V1
and, as the I/P current in Op-amp is always zero therefore, there will be no
voltage drop across 1 KW in II op-amp
i.e., V2 = 1 V
Therefore,

Downloaded From : www.EasyEngineering.net


Downloaded From : www.EasyEngineering.net

GATE SOLVED PAPER - EE 2013

V1 - V2 = V2 - ^- 2h
1 1
or, V1 - 1 = 1 + 2
or, V1 = 4
So,
Vout = 2V1 = 8 volt

Q. 35 In the circuit shown below, Q1 has negligible collector-to-emitter saturation


voltage and the diode drops negligible voltage across it under forward bias. If Vcc
is + 5 V , X and Y are digital signals with 0 V as logic 0 and Vcc as logic 1, then

t
the Boolean expression for Z is

e
.n
ng
ri
ee
in
(A) XY
(C) XY
ng (B) XY
(D) XY

ye
Sol. 35

as
Hence (B) is correct option
For the given circuit, we can make the truth table as below

.E
X Y Z
0 0 0

w 0 1 1

w 1
1
0
1
0
0
w
Logic 0 means voltage is v = 0 volt and logic 1 means voltage is 5 volt
For x = 0 , y = 0 , Transistor is at cut off mode and diode is forward biased. Since,
there is no drop across forward biased diode.
So, Z =Y=0
For x = 0 , y = 1, Again Transistor is in cutoff mode, and diode is forward biased.
with no current flowing through resistor.
So, Z =Y=1
For x = 1, y = 0 , Transistor is in saturation mode and so, z directly connected to
ground irrespective of any value of Y .
i.e., Z = 0 (ground)
Similarly for X = Y = 1
Z = 0 (ground)
Hence, from the obtained truth table, we get
Z =XY

Downloaded From : www.EasyEngineering.net


Downloaded From : www.EasyEngineering.net

GATE SOLVED PAPER - EE 2013

Q. 36 The clock frequency applied to the digital circuit shown in the figure below is
1 kHz. If the initial state of the output of the flip-flop is 0, then the frequency of
the output waveform Q in kHz is

et
.n
ng
(A) 0.25
(C) 1
ri (B) 0.5
(D) 2
Sol. 36

ee
Hence (B) is correct option

in
From the given logic diagram, we obtain
X = ^Q 5 Q h $ ^Q 5 Q h

ng =0=1
So, the input is always ‘1’ at T , since, clock is - ve edge trigged therefore, at
e
the negative edge Q changes its state as shown in waveform below
y
as
.E
w
w
w
Hence, as obtained from the waveform, time period of Q is double to that of
CLK I/p and so, frequency is 12 of clock frequency
Thus, fQ = FCLK = 1 = 0.5 kHz
2 2
z2 - 4 dz evaluated anticlockwise around the circle z - i = 2 , where i = - 1
Q. 37 # z2 + 4
, is

(A) - 4p (B) 0
(C) 2 + p (D) 2 + 2i
Sol. 37 Hence (A) is correct option
Given the contour integral
2
# zz2 -+ 44 dz
It has two poles given as
z = ! 2i
Now, the contour is defined by circle z - i = 2 which is shown in the figure
below

Downloaded From : www.EasyEngineering.net


Downloaded From : www.EasyEngineering.net

GATE SOLVED PAPER - EE 2013

So, it can be observed that the given contour enclosed z = 2i while z =- 2i is

2
residue = z - 4 t
out of the contour. So, we obtain the residue at z = 2i only as

e
z + 2i z = 2i

.n
^2i h2 - 4 - 8
=
g
2 i + 2i
n
=
4i
= 2i

# ri
Hence, contour integral is given as
z2 - 4 dz = 2pi (sum of residues)
z2 + 4

ee
= 2pi ^2i h

in =- 4p

Q. 38
1
ng
and > H respectively. The matrix is
1
A Matrix has eigenvalues - 1 and - 2 . The corresponding eigenvectors are > H
-1

(A) >
-2
1 1
- 1 - 2H ye (B) >
1 2
- 2 - 4H

as
.E
-1 0 0 1
(C) >
0 - 2H
(D) >
- 2 - 3H
Sol. 38

w
Hence (D) is correct option
We know that the characteristic equation is given by
w 6A@6X@ = l 6X@

w
where 6A@ is the matrix as l is the scalar which gives eigen values. Now, we
consider the matrix
a b
6A@ = >c dH ^2 # 2 matrixh
1
For eigen value - 1 as eigen vector is > H, so, we have
-1
a b 1 1
>c d H>- 1H =- 1 >- 1H
or, a - b =- 1 ....(1)
c-d = 1 ....(2)
1
Similarly, foreigen value - 2 with eigen vector > H, we obtain
-2
ab 1 1
>c H>
d -2 H =- 2 > H
-2
or, a - 2b =- 2 ....(3)
c - 2d = 4 ....(4)
Solving Eqs. (1) and (3), we obtain
a = 0, b = 1
and solving Eqs. (2) and (4), we obtain

Downloaded From : www.EasyEngineering.net


Downloaded From : www.EasyEngineering.net

GATE SOLVED PAPER - EE 2013

c =- 2, d = 3
Thus, the required matrix is
a b 0 1
>c d H = >- 2 - 3H
Q. 39 A dielectric slab with 500 mm # 500 mm cross-section is 0.4 m long. The slab
is subjected to a uniform electric field of Ev = 6avx + 8avy kV/mm . The relative
permittivity of the dielectric material is equal to 2. The value of constant e0 is
8.85 # 10-12 F/m . The energy stored in the dielectric in Joules is
(A) 8.85 # 10-11
(C) 88.5 et (B) 8.85 # 10-5
(D) 885
Sol. 39
.n
Hence (B) is correct option
g
Energy density stored in a dielectric medium is obtained as
n
ri
wE = 1 e E 2 J/m2
2

e
The electric field inside the dielectric will be same to given field in free space
e
n
only if the field is tangential to the interface
So,
gi wE = 1 2e0 ^ 62 + 82 h # 106 /mm2
2
2

# en
Therefore, the total stored energy is

y
WE = WE dv
v

a s = e0 100 # 106 /mm2 # ^500 # 500h mm2 # ^0.4h


= e0 # 100 # 106 # 0.4 # 25 # 10 4

.E = 8.85 # 10-12 # 1013

w = 88.5 J

w
For a power system network with n nodes, Z 33 of its bus impedance matrix is
w
Q. 40

j0.5 per unit. The voltage at node 3 is 1.3 - 10c per unit. If a capacitor having
reactance of - j3.5 per unit is now added to the network between node 3 and the
reference node, the current drawn by the capacitor per unit is
(A) 0.325 - 100c (B) 0.325 80c
(C) 0.371 - 100c (D) 0.433 80c
Sol. 40 Hence (D) is correct option
Q. 41 The separately excited dc motor in the figure below has a rated armature current
of 20 A and a rated armature voltage of 150 V. An ideal chopper switching
at 5 kHz is used to control the armature voltage. If La = 0.1 mH , Ra = 1 W ,
neglecting armature reaction, the duty ratio of the chopper to obtain 50% of the
rated torque at the rated speed and the rated field current is

(A) 0.4 (B) 0.5


(C) 0.6 (D) 0.7

Downloaded From : www.EasyEngineering.net


Downloaded From : www.EasyEngineering.net

GATE SOLVED PAPER - EE 2013

Sol. 41 Hence (D) is correct option


Given, the rated armature current
Ia^rated h = 20 A
as rated armature voltage
Va^rated h = 150 volt
Also, for the armature, we have
La = 0.1 mH , Ra = 1 W
and T = 50% of Trated ^T " Torqueh
So, we get
I = 6Ia^rotatedh@^0.5h = 10 A

et
N = Nrated ,
I f = I f rated " rated field current

n
At the rated conditions,

g.
E = V - Ia^ratedh Ra
= 150 - 20 ^1 h = 130 volt
For given torque,
in
V = E + Ia Ra = 130 + ^10h^1 h = 140 V
Therefore,
erchopper output = 140 V
or,

ne D ^200h = 140
or,

gi D = 140 = 0.7
200
(D " duty cycle)

Q. 42

en
A voltage 1000 sin wt Volts is applied across YZ . Assuming ideal diodes, the

y
voltage measured across WX in Volts, is

as
.E
w
w
w
(A) sin wt
(B) _sin wt + sin wt i /2
(C) ^sin wt - sin wt h /2
(D) 0 for all t
Sol. 42 Hence (D) is correct option
Given, the input voltage
VYZ = 100 sin wt

Downloaded From : www.EasyEngineering.net


Downloaded From : www.EasyEngineering.net

GATE SOLVED PAPER - EE 2013

et
For + ve half cycle
.n
ng
VYZ > 0
i.e., VY is a higher voltage than VZ

ri
So, the diode will be in cutoff region. Therefore, there will no voltage difference
between X and W node.
i.e.,
eeVWX = 0
Now, for - ve half cycle all the four diodes will active and so, X and W

in
terminal is short circuited
i.e.,
Hence,
ng VWX = 0
VWX = 0 for all t

ye
Three capacitors C1 , C2 and C 3 whose values are 10 mF , 5 mF , and 2 mF respectively,
s
Q. 43
have breakdown voltages of 10 V, 5 V and 2 V respectively. For the interconnection
a
shown below, the maximum safe voltage in Volts that can be applied across the

.E
combination, and the corresponding total charge in mC stored in the effective
capacitance across the terminals are respectively,
w
w
w
(A) 2.8 and 36 (B) 7 and 119
(C) 2.8 and 32 (D) 7 and 80
Sol. 43 Hence (C) is correct option

Consider that the voltage across the three capacitors C1 , C2 and C 3 are V1 , V2 and
V3 respectively. So, we can write
V2 = C 3 ....(1)
V3 C2
Since, Voltage is inversely proportional to capacitance
Now, given that

Downloaded From : www.EasyEngineering.net


Downloaded From : www.EasyEngineering.net

GATE SOLVED PAPER - EE 2013

C1 = 10 mF ; ^V1hmax = 10V
C2 = 5 mF ; ^V2hmax = 5 V
C 3 = 2 mF ; ^V3hmax = 2V
So, from Eq (1) we have
V2 = 2
V3 5
for ^V3hmax = 2
We obtain,
V2 = 2 # 2 = 0.8 volt < 5
5
V2 < ^V2hmax
t
i.e.,
Hence, this is the voltage at C2 . Therefore,
e
V3 = 2 volt
n
and
g.
V2 = 0.8 volt
V1 = V2 + V3 = 2.8 volt

n
Now, equivalent capacitance across the terminal is
i
erCeq = C2 C 3 + C1
C2 + C3
= 5 # 2 + 10

ne 5+2
80

gi =
7
mF
Equivalent voltage is (max. value)

en Vmax = V1 = 2.8
So, charge stored in the effective capacitance is

sy Q = Ceq Vmax

a = b 80 l # ^2.8h
7

.E = 32 mC

w
Q. 44
w
In the circuit shown below, if the source voltage VS = 100+53.13c V then the

w
Thevenin’s equivalent voltage in Volts as seen by the load resistance RL is

(A) 100+90c (B) 800+0c


(C) 800+90c (D) 100+60c
Sol. 44 Hence (C) is correct option
For evaluating the equivalent thevenin voltage seen by the load RL , we open the
circuit across it (also if it consist dependent source).
The equivalent circuit is shown below

Downloaded From : www.EasyEngineering.net


Downloaded From : www.EasyEngineering.net

GATE SOLVED PAPER - EE 2013

As the circuit open across RL so

or, et
I2 = 0
j40I2 = 0

.n
i.e., the dependent source in loop 1 is short circuited. Therefore,

g
VL1 =
n
^ j4h Vs
j4 + 3

ri
ee
VTh = 10 VL1 =
j40
j4 + 3
100 53.13c

n
40 90c
=
i
100 53.13c
5 53.13c

ng = 800 90c
w ^s h
= 10 . When
Q. 45

ye
The open-loop transfer function of a dc motor is given as
Va ^s h 1 + 10s
connected in feedback as shown below, the approximate value of Ka that will

as
reduce the time constant of the closed loop system by one hundred times as
compared to that of the open-loop system is

.E
w
w
w
(A) 1 (B) 5
(C) 10 (D) 100
Sol. 45 Hence (C) is correct option
Given, open loop transfer function
G ^s h = 10Ka = Ka 1
1 + 10s s + 10
By taking inverse Laplace transform, we have
g ^ t h = e- t
1
10

Comparing with standard form of transfer function, Ae-t/t , we get the open
loop time constant,
tol = 10
Now, we obtain the closed loop transfer function for the given system as
G ^s h
H ^s h = = 10Ka
1 + G ^s h 1 + 10s + 10Ka
= Ka
s + ^Ka + 101 h
By taking inverse laplace transform, we get
h ^ t h = ka .e-^k + ht
1
a 10

So, the time constant of closed loop system is obtained as

Downloaded From : www.EasyEngineering.net


Downloaded From : www.EasyEngineering.net

GATE SOLVED PAPER - EE 2013

tcl = 1
ka + 101
or, tcl = 1 (approximately)
ka
Now, given that ka reduces open loop time constant by a factor of 100. i.e.,
tcl = tol
100
or, 1 = 10
ka 100
or, ka = 10
Q. 46 In the circuit shown below, the knee current of the ideal Zener dioide is 10 mA

et
. To maintain 5 V across RL , the minimum value of RL in W and the minimum
power rating of the Zener diode in mW, respectively, are

.n
ng
ri
ee
in
ng
(A) 125 and 125 (B) 125 and 250

ye
(C) 250 and 125 (D) 250 and 250
Sol. 46

as
Hence (B) is correct option

.E
w
w
w
From the circuit, we have
Is = I Z + I L
or, I Z = Is - I L (1)
Since, voltage across zener diode is 5 V so, current through 100 W resistor is
obtained as
Is = 10 - 5 = 0.05 A
100
Therefore, the load current is given by
IL = 5
RL
Since, for proper operation, we must have
IZ $ Iknes
So, from Eq. (1), we write
0.05 A - 5 $ 10 mA
RL

Downloaded From : www.EasyEngineering.net


Downloaded From : www.EasyEngineering.net

GATE SOLVED PAPER - EE 2013

50 mA - 5 $ 10 mA
RL
40 mA $ 5
RL
40 # 10 $ 5
-3
RL
1 # RL
40 # 10-3 5

or,
5
40 # 10-3
et# RL
125 W # RL

.n
Therefore, minimum value of RL = 125 W

ng
Now, we know that power rating of Zener diode is given by
PR = VZ IZ^maxh

ri
IZ^maxh is maximum current through zener diode in reverse bias. Maximum

ee
currrent through zener diode flows when load current is zero. i.e.,
IZ^maxh = Is = 10 - 5 = 0.05
100
Therefore,
in PR = 5 # 0.05 W

Q. 47
ng = 250 mW
A strain gauge forms one arm of the bridge shown in the figure below and has

ye
a nominal resistance without any load as Rs = 300 W . Other bridge resistances
are R1 = R2 = R 3 = 300 W . The maximum permissible current through the strain

as
gauge is 20 mA. During certain measurement when the bridge is excited by
maximum permissible voltage and the strain gauge resistance is increased by 1%

.E
over the nominal value, the output voltage V0 in mV is

w
w
w

(A) 56.02 (B) 40.83


(C) 29.85 (D) 10.02
Sol. 47 Hence (C) is correct option

Common Data Questions: 48 & 49


The state variable formulation of a system is given as
xo1 - 2 0 x1 1 x1
> H=> H> H + > H u , x1 ^0 h = 0 , x2 ^0 h = 0 and y = 61 0@> H
xo2 0 - 1 x2 1 x2

Q. 48 The response y ^ t h to the unit step input is


(A) 1 - 1 e-2t (B) 1 - 1 e-2t - 1 e-t
2 2 2 2
(C) e-2t - e-t (D) 1 - e-t

Downloaded From : www.EasyEngineering.net


Downloaded From : www.EasyEngineering.net

GATE SOLVED PAPER - EE 2013

Sol. 48 Hence (A) is correct option


Given, the state variable formulation,
xo1 - 2 0 x1 1
> o H = > 0 - 1H>x H + >1H u ....(1)
x2 2
x1
and y = 61 0@> H ....(2)
x2
From Eq. (1) we get
xo1 = 2x1 + u
Taking Laplace transform
sX1 - x1 ^0 h =- 2X1 + 1 (Here, X1 denotes Laplace transform of x1 )
So,
e
^s + 2h X1 = s1
t s

^x1 ^0 h = 0h

.n 1
or,

ng
X1 =
s ^s + 2h
....(3)

i
Now, from Eq. (2) we have

er
y = x1
Taking Laplace transform both the sides,

ne
Y = XL
Y = 1

i
or, (from eq. (3))
s ^s + 2h
or,
ng Y = 1 ;1 - 1 E
2 s s+2

ye
Taking inverse Laplace transform
y = 1 8u ^ t h - e-2t u ^ t hB
2

as = - 1 e-2t
1
2 2 ^for t > 0h

.E
Q. 49 The system is
(A) controllable but not observable

w
(B) not controllable but observable

w
(C) both controllable and observable

Sol. 49
w
(D) both not controllable and not observable
Hence (A) is correct option
From the given state variable system, we have
-2 0
A =>
0 1H
1
and B = > H; C = 61 0@
1
Now, we obtain the controllability matrix
CM = 6B : AB@
1 -2
=>
2 1H
and the observability matrix is obtained as
C
OM = > H
CA
1 0
=> H
-2 0
So, we get

Downloaded From : www.EasyEngineering.net


Downloaded From : www.EasyEngineering.net

GATE SOLVED PAPER - EE 2013

Rank of the controllability matrix " Rank ^CM h = 2


Rank of the observability matrix " Rank ^OM h = 1
Since, the order of state variable is 2 ^x1 and x2h. Therefore, we have
Rank ^CM h = order of state variables
but, Rank (OM ) < order of state variables
Thus, system is controllable but not observable

et
Common Data Questions: 50 & 51
In the figure shown below, the chopper feeds a resistive load from a battery

.n
source. MOSFET Q is switched at 250 kHz, with duty ratio of 0.4. All elements

g
of the circuit are assumed to be ideal

n
ri
ee
in
ng
Q. 50
ye
The Peak to Peak source current ripple in amps is
(A) 0.96

as
(C) 0.192
(B) 0.144
(D) 0.228
Sol. 50
.E
Hence (C) is correct option

w
Here, as the current from source of 12 V is the same as that pass through inductor.
So, the peak to peak current ripple will be equal to peak to peak inductor current.
w
Now, the peak to peak inductor current can be obtained as

wIL (Peak to Peak) = Vs D Ts

where,
L
Vs " source voltage = 12 volt ,
L " inductance = 100mH = 10-4 H ,
D " Duty ration = 0.4 ,
TS " switching time period of MOSFET = 1
fS
and fs " switching frequency = 250 kHz
Therefore, we get
IL^Peak to Peakh = 12-4 # 0.4 # 1
10 250 # 103
= 0.192 A
This is the peak to peak source current ripple.
Q. 51 The average source current in Amps in steady-state is
(A) 3/2 (B) 5/3
(C) 5/2 (D) 15/4
Sol. 51 Hence (B) is correct option
Here, the average current through the capacitor will be zero. (since, it is a boost
converter). We consider the two cases :

Downloaded From : www.EasyEngineering.net


Downloaded From : www.EasyEngineering.net

GATE SOLVED PAPER - EE 2013

Case I : When MOSFET is ON


ic =- i 0
1
(i 0 is output current)
(since, diode will be in cut off mode)
Case II : When MOSFET is OFF
Diode will be forward biased and so
ic = Is - i 0
1
(Is is source current)
Therefore, average current through capacitor
i + Ic
Ic, avg = c 1 2

2
DTs ^- io h + ^1 - D h Ts ^Is - io h
& 0= (D is duty ratio)
2

et
Solving the equation, we get
Is =
^
i0
Dh
....(1)

.n 1 -
Since, the output load current can be given as

ng V/
i 0 = V0 = s 1 - D =
12/0.6
= 1A

i
R R 20

er
Hence, from Eq. (1)
Is = i 0 = 1 = 5 A
1-D
e
0.6 3

in
Linked Answer Questions: Q.52 to Q.55 Carry Marks Each

ng
ye
Statement for Linked Answer Questions: 52 & 53
In the following network, the voltage magnitudes at all buses are equal to 1 pu,

as
the voltage phase angles are very small, and the line resistances are negligible. All
the line reactances are equal to j1 W

.E
w
w
w
Q. 52 The voltage phase angles in rad at buses 2 and 3 are
(A) q2 =- 0.1, q3 =- 0.2 (B) q2 = 0 , q3 =- 0.1
(C) q2 = 0.1, q3 = 0.1 (D) q2 = 0.1, q3 = 0.2
Sol. 52 Hence (C) is correct option
Consider the voltage phase angles at buses 2 and 3 be Q2 and Q 3 since, all the
three buses have the equal voltage magnitude which is 1 Pu so, it is a D.C. load
flow. The injections at Bus 2 and 3 are respectively
P2 = 0.1 pu
P3 =- 0.2 pu
Therefore, the phase angles are obtained as
q2 P2
>q H = 6B@-1 >P H
3 3

Downloaded From : www.EasyEngineering.net


Downloaded From : www.EasyEngineering.net

GATE SOLVED PAPER - EE 2013

where 6B@ is obtained as


R V
S 1 + 1 1 W
X12 X23 X23 W
6B@ = S - 1
S
1 + 1 W
S X23 X23 X13 W
T X
1 + 1 -1 2 -1
=> H=> H
-1 1 + 1 -1 2
Its inverse is obtained as

t 2 -1
6B@ = >1 - 2 H
-1

e
-1

.n= 1>
2 1 0. 1
H> H

g
3 + 1 2 - 0. 2

n
Therefore,

riq2 1 2 1 0. 1
>q H = 3 >+ 1 2H>- 0.2H

e
3
0

ne = 1>
3 - 0.3
H

gi =>
0
- 0.1H
i.e.,
and
en q2 = 0
q3 =- 0.1 rad
Q. 53
sy
If the base impedance and the line-to line base voltage are 100 ohms and 100 kV

a
respectively, then the real power in MW delivered by the generator connected at

.E
the slack bus is
(A) - 10 (B) 0

w
(C) 10 (D) 20
Sol. 53
w
Hence (C) is correct option
From the above solution, we have
w P2 = 0.1
P3 =- 0.2
since,
P1 + P2 + P3 = 0 (Where P1 is injection at bus 1)
So, P1 - P2 - P3 =- 0.1 + 0.2 = 0.1 pu
Now, the apparent power delivered to base is
^100 # 10 h
2 3 2
S = V =
R 100
6
= 100 # 10 VA
Therefore, the real power delivered by slack bus (bus 1)
P = P1 S = ^0.1h # 100 # 106
= 10 # 106 watt
= 10 MW

Statement for Linked Answer Questions: 54 & 55


The Voltage Source Inverter (VSI) shown in the figure below is switched to
provide a 50 Hz, square wave ac output voltage Vo across an RL load. Reference
polarity of Vo and reference direction of the output current io are indicated in the
figure. It is given that R = 3 ohms, L = 9.55 mH .

Downloaded From : www.EasyEngineering.net


Downloaded From : www.EasyEngineering.net

GATE SOLVED PAPER - EE 2013

Q. 54

current is
et
In the interval when V0 < 0 and i 0 > 0 the pair of devices which conducts the load

(A) Q1, Q2
.n
(B) Q 3, Q 4
(C) D1, D2
ng
(D) D 3, D 4
ri
Sol. 54

ee
Hence (D) is correct option
We consider the following two cases :

in
Case I : When Q1, Q2 ON

ng
In this case the + ve terminal of V0 will be at higher voltage. i.e. V0 > 0 and so
i 0 > 0 (i.e., it will be + ve ). Now, when the Q1 , Q2 goes to OFF condition we

ye
consider the second case.
Case II : When Q 3 , Q 4 ON and Q , Q2 OFF :

as
In this condition, - ve terminal of applied voltage V0 will be at higher potential
i.e., V0 < 0 and since, inductor opposes the change in current so, although the

.E
polarity of voltage V0 is inversed, current remains same in inductor i.e. I 0 > 0 .
This is the condition when conduction have been asked.
w In this condition ^V0 > 0, I 0 > 0h since, IGBT’s can’t conduct reverse

w
currents therefore current will flow through D 3, D 4 until ID becomes negative.

w
Thus, D 3 and D 4 conducts.

Q. 55 Appropriate transition i.e., Zero Voltage Switching ^ZVS h/Zero Current Switching
^ZCS h of the IGBTs during turn-on/turn-off is
(A) ZVS during turn off
(B) ZVS during turn-on
(C) ZCS during turn off
(D) ZCS during turn-on

Sol. 55 Hence (D) is correct option


When Q 3, Q 4 is switched ON, initially due to the reverse current it remain in OFF
state and current passes through diode. In this condition the voltage across Q 3
and Q 4 are zero as diodes conduct. Hence, it shows zero voltage switching during
turn-ON

Downloaded From : www.EasyEngineering.net


Downloaded From : www.EasyEngineering.net

GATE SOLVED PAPER - EE 2013

General Aptitude (GA) Questions

Q.56 to Q.60 carry one mark each.


Q. 56 Choose the grammatically CORRECT sentence:
(A) Two and two add four
(B) Two and two become four
(C) Two and two are four

et
(D) Two and two make four
Sol. 56
.n
Hence (D) is correct option
g
Two and two make four
n
Q. 57

ri
Statement: You can always give me a ring whenever you need.

ee
Which one of the following is the best inference from the above statement?
(A) Because I have a nice caller tune.

n
(B) Because I have a better telephone facility
i
g
(C) Because a friend in need is a friend indeed

ring
en
(D) Because you need not pay towards the telephone bills when you give me a

Sol. 57

sy
Hence (C) is correct option
You can always given me a ring whenever you need. Because a friend is need is a
a
friend indeed
Q. 58
.E
In the summer of 2012, in New Delhi, the mean temperature of Monday to

w
Wednesday was 41°C and of Tuesday to Thursday was 43cC . If the temperature
on Thursday was 15% higher than that of Monday, then the temperature in cC

w
on Thursday was

w
(A) 40
(C) 46
(B) 43
(D) 49
Sol. 58 Hence (C) is correct option
Let the temperature on Monday, Tuesday, Wednesday and Thursday be respectively
as TM , TTU , TW , TTH
So, from the given data we have
TH + TTU + TW = 41 ....(1)
3
and TTU + TW + TTH = 43 ....(2)
3
also, as the temperature on Thursday was 15% higher than that of Monday
i.e. TTH = 1.15 TM ....(3)
solving eq (1), (2) and (3), we obtain
TTH = 46cC
Q. 59 Complete the sentence: Dare . ................ mistakes.
(A) commit (B) to commit
(C) committed (D) committing

Downloaded From : www.EasyEngineering.net


Downloaded From : www.EasyEngineering.net

GATE SOLVED PAPER - EE 2013

Sol. 59 Hence (B) is correct option


Dare to commit mistakes

Q. 60 They were requested not to quarrel with others.


Which one of the following options is the closest in meaning to the word quarrel?
(A) make out (B) call out
(C) dig out (D) fall out
Sol. 60 Hence (D) is correct option
They were requested not to quarrel with others.
Quarrel has a similar meaning to ‘fall out’

et
.n
Q. No. 61-65 Carry Two Marks Each
Q. 61

ng
A car travels 8 km in the first quarter of an hour, 6 km in the second quarter and

i
16 km in the third quarter. The average speed of the car in km per hour over the
entire journey is
(A) 30
er (B) 36
(C) 40

ne (D) 24
Sol. 61

gi
Hence (C) is correct option
Given, the distance travelled by the car in each quarter intervals as

en Distance Time Duration

y 8 km 1
4 hr

as 6 km
16 km
1
4
1
hr
hr

.E
4

Therefore, the total time taken = 1 + 1 + 1 + 3 hr


4 4 4 4
w Total distance travelled = 8 + 6 + 16 = 30 km

w
Hence, average speed = Total distance travelled
Total time taken
w = 30
3/4
= 40 km/hr
Q. 62 Find the sum to n terms of the series 10 + 84 + 734 + ...
9 ^9n + 1h 9 ^9n - 1h
(A) +1 (B) +1
10 8
9 ^9n - 1h 9 ^9n - 1h
(C) +n (D) + n2
8 8
Sol. 62 Hence (D) is correct option
It will be easy to check the options for given series. From the given series.
10 + 84 + 734 + . ....
We get
Sum of 1 term = S1 = 10
Sum of 2 terms = S2 = 10 + 84 = 94
and sum of 3 terms = S 3 = 10 + 84 + 734 = 828
Checking all the options one by one, we observe that only (D) option satisfies as

Downloaded From : www.EasyEngineering.net


Downloaded From : www.EasyEngineering.net

GATE SOLVED PAPER - EE 2013

9 ^9n - 1h
Sn = + n2
8
9 ^92 - 1h
so, S1 + 22 = 10
8
9 ^9 - 1h
S2 = + 22 = 94
8
9 ^93 - 1h
S3 = + 32 = 828
8
Q. 63

et
Statement: There were different streams of freedom movements in colonial India
carried out by the moderates, liberals, radicals, socialists, and so on.

.n
Which one of the following is the best inference from the above statement?

g
(A) The emergence of nationalism in colonial India led to our Independence

n
(B) Nationalism in India emerged in the context of colonialism

i
er
(C) Nationalism in India is homogeneous
(D) Nationalism in India is heterogeneous
Sol. 63

ne
Hence (D) is correct option

Q. 64
gi
Nationalism in India is heterogeneous

The set of values of p for which the roots of the equation 3x2 + 2x + p ^p - 1h = 0

en
are of opposite sign is
(A) ^- 3, 0h (B) ^0, 1h

sy
(C) ^1, 3h (D) ^0, 3h
Sol. 64
a
Hence (B) is correct option

.E
Given, the quadratic equation
3x2 + 2x + P ^P - 1h = 0
w
It will have the roots with opposite sign if

w P ^P - 1h < 0

w
So it can be possible only when
P < 0 and P - 1 > 0
or P > 0 and P - 1 < 0
st
The 1 condition tends to no solution for P .
Hence, from the second condition, we obtain
0 <P<1
i.e., P is in the range ^0, 1h
Q. 65 What is the chance that a leap year, selected at random, will contain 53 Sundays?
(A) 2/7 (B) 3/7
(C) 1/7 (D) 5/7
Sol. 65 Hence (A) is correct option
In a leap year, there are 366 days So, 52 weeks will have 52 saturdays and for
remaining two days ^366 - 52 # 7 = 2h. We can have the following combinations
Saturday, Sunday
Sunday, Monday
Monday, Tuesday
Tuesday, Wednesday
Wednesday, Thursday

Downloaded From : www.EasyEngineering.net


Downloaded From : www.EasyEngineering.net

GATE SOLVED PAPER - EE 2013

Thursday, Friday
Friday, Saturday
Out of these seven possibilities, only two consist a saturday. Therefore, the
probability of saturday is given as
P =2
7

et
.n
ng
ri
ee
in
ng
ye
as
.E
w
w
w

Downloaded From : www.EasyEngineering.net


Downloaded From : www.EasyEngineering.net

GATE SOLVED PAPER - EE 2013

Answer Sheet
1. (B) 13. (C) 25. (A) 37. (C) 49. (B) 61. (B)
2. (A) 14. (C) 26. (D) 38. (C) 50. (D) 62. (D)
3. (D) 15. (*) 27. (C) 39. (D) 51. (C) 63. (D)
4. (B) 16. (A) 28. (B) 40. (C) 52. (B) 64. (B)
5.
6.
(B)
(*)
17.
18.
(A)
(C)
29.
30. et (B)
(C)
41.
42.
(*)
(B)
53.
54.
(D)
(*)
65. (C)

7. (B) 19. (C) 31.


.n (D) 43. (A) 55. (*)
8. (D) 20. (B)
ng
32. (C) 44. (B) 56. (D)
9. (D) 21. (A)
ri
33. (B) 45. (A) 57. (B)
10.
11.
(A)
(C)
22.
23.
(B)
(A)
ee 34.
35.
(C)
(A)
46.
47.
(A)
(D)
58.
59.
(D)
(B)
12. (*) 24. (C)
in 36. (A) 48. (B) 60. (C)

g
**********

en
sy
a
.E
w
w
w

Downloaded From : www.EasyEngineering.net

You might also like